Find the slope of the line passing through the point (-6,7) and (-6,-8).

Answers

Answer 1

The gradient/slope is undefined.

using the gradient formula, [tex]m=\frac{y2-y1}{x2-x1}[/tex],

[tex]\frac{-8-7}{-6-(-6)}[/tex]

try solving this and you'll get the answer "math error". this is because -6-(-6) is -6+6, which equals 0. anything divided by zero is infinite.


Related Questions

A soccer team won only 10 of their first 30 games how many of the remaining 20 games will they have to win to have a .500 record?

Answers

Answer:

They would have to win 15 games to have a .500 record.

Step-by-step explanation:

I'm pretty sure that I'm correct. Hope this helps and have a nice day!

Answer:

they have to win 15 games more to have a .500 record

Step-by-step explanation:

If .500 means half,then :

half of 50 games= 25 games

They already won 10

So they need to win 15 more games to have 25 games (half of the 50 games)

How do i answer this I am confused ​

Answers

Answer:

C

Step-by-step explanation:

this is the only equation that has a solution of (0,0) which means, when graphed, the line passes through the point of origin

answer A would cross the y-axis at -1/2, not 0

answer B would cross the y-axis at -2, not 0

answer D is not a linear equation

find the value of :




Answers

ANSWER:

5

hope it's helpful ❤❤❤

THANK YOU.

Diva wants to make a flower arrangement for her aunt's birthday. She wants of the
arrangement to be roses. She has 12 roses. How many other flowers does she need to
finish the arrangement?

Answers

Answer:

This makes no sense.

Step-by-step explanation:

There can’t be an answer, there’s no other numbers involved.

Convert the measurement. HELP ME ASAP

Answers

Answer:

1800 cm

Step-by-step explanation:

The price of a t-shirt is 50$. If you buy 4 shirts, you will get 10% discount. How much do
you have to pay if you buy 4 shirts?

Answers

Answer:

180

Step-by-step explanation:

Does anyone know the answer for this question ​

Answers

Uhm I think it’s 3.6

Work out $1.20 as a percentage of $16

Answers

Answer:

answer is 7.5

hopes this helps

Answer:

7.5%

Step-by-step explanation:

1.2 divided by 16 equals 0.075

then multiply by 100 equals 7.5%

Help.
Please its urgent show workings.

Answers

Answer:

see explanation

Step-by-step explanation:

There are 2 possible approaches to differentiating these.

Expand the factors and differentiate term by term, or

Use the product rule for differentiation.

I feel they are looking for use of product rule.

Given

y = f(x). g(x) , then

[tex]\frac{dy}{dx}[/tex] = f(x).g'(x) + g(x).f'(x) ← product rule

(a)

y = (2x - 1)(x + 4)²

f(x) = 2x - 1 ⇒ f'(x) = 2

g(x) = (x + 4)²

g'(x) = 2(x + 4) × [tex]\frac{d}{dx}[/tex] (x + 4) ← chain rule

       = 2(x + 4) × 1

        = 2(x + 4)

Then

[tex]\frac{dy}{dx}[/tex] = (2x - 1). 2(x + 4) + (x + 4)². 2

    = 2(2x - 1)(x + 4) + 2(x + 4)² ← factor out 2(x + 4) from each term

    = 2(x + 4) (2x - 1 + x + 4)

    = 2(x + 4)(3x + 3) ← factor out 3

    = 6(x + 4)(x + 1)

--------------------------------------------------------------------------

(b)

y =  x(x² - 1)³

f(x) = x ⇒ f'(x) = 1

g(x) = (x² - 1)³

g'(x) = 3(x² - 1)² × [tex]\frac{d}{dx}[/tex] (x² - 1) ← chain rule

        = 3(x² - 1)² × 2x

        = 6x(x² - 1)²

Then

[tex]\frac{dy}{dx}[/tex] = x. 6x(x² - 1)² + (x² - 1)³. 1

    = 6x²(x² - 1)² + (x² - 1)³ ← factor out (x² - 1)²

    = (x² - 1)² (6x² + x² - 1)

     = (x² - 1)²(7x² - 1)

----------------------------------------------------------------------

(c)

y = (x² - 1)(x³ + 1)

f(x) = x² - 1 ⇒ f'(x) = 2x

g(x) = (x³ + 1) ⇒ g'(x) = 3x²

Then

[tex]\frac{dy}{dx}[/tex] = (x² - 1). 3x² + (x³ + 1), 2x

   = 3x²(x² - 1) + 2x(x³ + 1) ← factor out x

   = x[3x(x² - 1) + 2(x³ + 1) ]

   = x(3x³ - 3x + 2x³ + 2)

   = x(5x³ - 3x + 2) ← distribute

    = 5[tex]x^{4}[/tex] - 3x² + 2x

--------------------------------------------------------------------

(d)

y = 3x³(x² + 4)²

f(x) = 3x³ ⇒ f'(x) = 9x²

g(x) = (x² + 4)²

g'(x) = 2(x² + 4) × [tex]\frac{d}{dx}[/tex](x² + 4) ← chain rule

       = 2(x² + 4) × 2x

       = 4x(x² + 4)

Then

[tex]\frac{dy}{dx}[/tex] = 3x³. 4x(x² + 4) + (x² + 4)². 9x²

    = 12[tex]x^{4}[/tex](x² + 4) + 9x²(x² + 4)² ← factor out 3x²(x² + 4)

    = 3x²(x² + 4) [ 4x² + 3(x² + 4) ]

    = 3x²(x² + 4)(4x² + 3x² + 12)

    = 3x²(x² + 4)(7x² + 12)

I CAN GIVE BRAINLIEST PLEASE HELP
Find the measure of angle x.

A.34

B.40

C.56

D.59

Answers

x = inverse of sine(24/29)
x = 55.8 or 56 so the answer’s C

Answer:

It is 56 exactly 55.8

Step-by-step explanation:

plssssssssss mark brainliest

What does nine times twelve equal to

Answers

Answer:

108

Step-by-step explanation:

9 x 12 = 108

Check

108 / 12 = 9

108/9 = 12

6. Which exbression is equivalent to 24x2 - 22x + 5?
A (12x + 5)(2x + 1)
B (8x - 5)(3x - 1)
C (12x - 5)(2x - 1)
D (8x + 5)(3x + 1)

Answers

Answer: A (12x−5)(2x−1)

Step-by-step explanation:

The expression which is equivalent to the given would be; C (12x - 5)(2x - 1).

What are equivalent expressions?

Those expressions that might look different but their; simplified forms are the same expressions are called equivalent expressions.

To derive equivalent expressions of some expressions, we can either make it look more complex or simple.

We have been given an expression as;

24x² - 22x + 5

Factor out;

24x² - (12 + 10)x + 5

(12x−5)(2x−1)

Therefore, the expression which is equivalent to the given would be; C (12x - 5)(2x - 1).

Learn more about expression here;

https://brainly.com/question/14083225

#SPJ6

Can someone help me with one of these :>

Answers

13) 180 degrees = 130 degrees + 3x + 2x
180 degrees = 130 degrees + 5x
subtract 130 from both sides to isolate the 5x
50 degrees = 5x
divide both sides by 5 to isolate the x
10 degrees = x
3(10) = 30 degrees
A = 30 degrees

If there are two angles that are complementary to each other and one of the angles is 48 degrees, what is the other angle?

Answers

Answer:

42 degrees

Step-by-step explanation:

Please let me know if you want me to add an explanation as to why this is the answer. I can definitely do that, I just don’t want to waste my time in case you don’t want me to :)

Which equation represents an ellipse? Help please!

Answers

Answer:

2

Step-by-step explanation:

2

Answer: A

Step-by-step explanation:

PLEASE HELP ASAP!!
Candidate A receives 3 votes for every 1 vote that candidate B receives. At the end of the election candidate B has 900 votes. Candidate A recieved votes.

Answers

Answer:

2700 because

Step-by-step explanation:

I multiplied 900 by 3

Answer:

2700

Step-by-step explanation:

Let m = 14 and n = 12 m + 11 + 2n

Answers

Answer:

79

Step-by-step explanation:14+11+2(12) = 24 + 14 + 11 = 79

Answer:

49

Step-by-step explanation:

14+11+2(12)

14+11+24

25+24

49

find the sum of the first 50 terms.
-6,-2,2,6

Answers

Answer:

4600

Step-by-step explanation:

There is a common difference d between consecutive terms, that is

d = - 2 - (- 6) = 2 - (- 2) = 6 - 2 = 4

This indicates the sequence is arithmetic with sum to n terms

[tex]S_{n}[/tex] = [tex]\frac{n}{2}[/tex] [ 2a₁ + (n - 1)d ]

where a₁ is the first term and d the common difference

Here a₁ = - 6 and d = 4 , then

[tex]S_{50}[/tex] = [tex]\frac{50}{2}[/tex] [ (2 × - 6) + (49 × 4) ]

     = 25(- 12 + 196)

     = 25 × 184

     = 4600

The sum of the first 50 terms of the sequence -6,-2,2,6,...., is 4600.

What is an arithmetic sequence?

A series of integers called an arithmetic progression or arithmetic sequence (AP) has a constant difference between the terms.

We have the sequence:

-6,-2,2,6,....

In order to solve the problem:

First, we find the common difference.

-2 + 6 = 4,

2 + 2 = 4,

6 - 2= 4.

d = 4 is the common difference.

That means, the sequence is an arithmetic sequence.

The sum of n terms,

=  [2a₁ + (n - 1)d],

where a₁ is the first term and d is the common difference.

Here, a₁ = - 6, n = 50 and d = 4,

Then

[2a₁ + (n - 1)d] =  [(2 × -6) + (49 × 4)]

                     = 25(- 12 + 196)

                     = 25 × 184

                     = 4600.

Therefore, the sum of the first 50 term is 4600.

To learn more about the arithmetic sequence;

https://brainly.com/question/29616017

#SPJ2

Logan is making bookmarks that contain flower petals. He has 95 petals. He uses 3 flower petals for each bookmark. What is the GREATEST number of bookmarks that Logan can make?

Answers

Answer: The GREATEST number of bookmarks that Logan can make = 31.

Step-by-step explanation:

GIven: Total petals = 95

Number of petals required for each bookmark = 3

Number the number of bookmarks can be prepared from 95 petals = 95 ÷ (Petals required of each bookmark)

= 95÷3

[tex]=31\dfrac{2}{3}\ \ \ \approx31\text{ [Round to the nearest whole number.]}[/tex]

i.e. The GREATEST number of bookmarks that Logan can make = 31.

Susan claims. "If a family has already got four boys, then the next baby is more likely to be a girl than a boy."

Answers

Answer:

Assuming that for every baby the probability of being a girl or a boy is the same (50% for each of them)

We could then think in this situation as a coin flip.

Suppose that you flip a coin 4 times, and the four times you get heads.

Does this imply that in the 5th flip the probability of getting tails increments?

Well, not, each flip is an individual event, and it's outcome does not depend on the outcome of the previous flips.

The same is for the new baby, the gender does not depend on the gender of the previous babies, is an independent event, then Susan's statement is false.

please help me with this it's a huge test :) i'll mark brainliest!

Answers

Answer:

I think A or D

Step-by-step explanation:

f (x) =3x + 4, f(x) = -20

Answers

Answer: x= -8 and y = -20

3. Andrea is buying snacks at the basketball game. The snacks are $2.25 apiece. She wants to buy 4
snacks. Would the ordered pair (4,9.25) be a solution to this problem? Explain why or why not.
If this is not a solution, provide the correct solution for an x-value of 4.
a) Yes
b) No, the ordered pair (9,4) is a solution to this problem.
c) No, the ordered pair (4, 1.77) is a solution to this problem
d) No, the Ordered pair (4,9) is a solution to this problem.

Answers

Answer:

D - No, the ordered pair (4,9) is a solution to this problem.

Step-by-step explanation:

If you plotted the number of snacks as the x-axis, and then the total cost of the snacks on the y-axis, you would be able to graph a line and see the cost of snacks based on the number purchased. If you were going to write an equation for this, it would be y=2.25x (or y = 2 1/4 x)

In this case, as each snack is the same price, the y-values for each x would be as such: (1, 2.25) <- one snack, $2.25;

(2, 4.5)

(3, 6.75)

(4, 9)

So that is why the ordered pair of (4,9) would be a solution to this problem.

Hope that makes sense!

If a || b and ____, then a || c.

• b || c
• a upside down T c
• b upside down T c
• b upside down T a

Answers

Answer:

it will option A it is correct answer

34 + 2 ⋅ 5 =
I really need help with this

Answers

Answer:

44

Step-by-step explanation:

PEMDAS

34+2*5

34+10

44

Answer: 36.5



Explanation

Suppose you want to make a rectangle with perimeter 400. What dimensions give you
the maximum area?
I

Answers

Answer:

100X100

Step-by-step explanation:

sorry if i get it wrong, hope this helps!

i will give anyone brainliest thats writes down the correct answer for rhis question Layla is using the model to solve a division problem.

An area model has 4 shaded parts and 2 unshaded parts. The shaded part is labeled two-thirds.
Which question would the model be most helpful in solving?
How many One-sixths are in 4?
How many One-sixths are in Two-thirds?
How many Two-thirdss are in 4?
How many Two-thirdss are in One-sixth?

Answers

Answer:

How many One-sixths are in Two-thirds?

Step-by-step explanation:

Answer:  How many One-sixths are in Two-thirds?

Step-by-step explanation: please mark me as a brainly

Is-5 a solution to the equation 6x+5=5x+8+2 x? Explain. please help asap

Answers

Answer:

6x+5=5x+8+2x

x= -3

Step-by-step explanation:

use like terms so add 5x+2x=7x

6x+5=7x+8

subtract 5 from both sides

6x+5-5=7x+8-5

simplify

-x = 3

divide both sides by -1

-x/-1  =  3/-1

x= -3

so no -5 is not a solution

can anyone answer -4(3-x)-7x and show the work !

Answers

the answer is -12-3x

Given the equation y= -3x + 2.5 . When x is 0, what value of y makes the equation true?

Answers

Answer:

2.5

Step-by-step explanation:

plug in 0 where x is in the equation to find y

y = -3x + 2.5

y = -3(0) + 2.5

y = 2.5

check it by subbing 2.5 into the y values

2.5 = -3x + 2.5

-2.5. -2.5

0 = -3x

+3x

3x = 0

x = 0

Other Questions
Choose the most relevant and credible evidence for this claim:Students should perform Shakespeare plays using the originallanguage, not "easier" modernized versions of the play.A. After the Maplewood Middle School Drama Club performedscenes from Macbeth, 94 percent of students reported that theyfelt that speaking Shakespeare's original language had been avaluable experience.B. According to Shakespeare scholar Juliet Rosalind, Shakespeareanlanguage is difficult for students not only because the wordsthemselves are unfamiliar but because they describe ideas andsituations that reflect Shakespeare's time, not our own.C. I read my first original Shakespeare play in sixth grade, and while itwas difficult and I didn't understand everything, the experience didnothing but increase my love for the theater.D. The themes of Shakespeare's plays are surprisingly mature, andmost of them need to be edited in order to be made appropriatefor a school setting. 3 + 9y 5z = -5 3 + 2y 3z = 93 + + 4z = 7 Complete the statement for the equation y=-1/2x-1/2. Enter the correct answer in the box.The graph crosses the y-axis at (0,?) Noah says that 9X minus 2X plus 4X is equivalent to 3X because the subtraction sign tells us to subtract everything that comes after 9X Which of the following is equivalent to theexpression above?A) 5x2 - 2x - 3B) 5x + 2x - 3C) 5x + 2x + 3 D) 5x2 + 12x + 15 When coach Gomez held tryouts for the track team, 40 students tried out.only 25 of the students made the team. what percentage of the students made the team Find x and y (Pythagorean Theorem) An = a1 + (n - 1)dA car travels 300 meters the first minute. Each minute after, the car travels 120 meters. If this pattern continues, what isthe distance the car travels in 15 minutes.4320 metersO 2000 metersO 2085 metersO 1980 meters What is 3785 divided by 30? A single die is rolled twice find the probability of rolling a 1 the first time and a 6 the second time. Answer the question using evidence to support your answer. (AT LEAST 5 SENTENCES) Question: WHAT WERE SOME OF THE RESULTS (EFFECTS) OF THE SEVEN YEARS WAR?Things to consider: Land (In North America, Canada, India, The Caribbeans, etc) Native Americans British population and power British Debt Solve the equation and check your solution. (If an answer does not exist, enter DNE. If all real numbers are solutions, enter REALS.)2 4x = 34 A base produces OH-1 ions when dissolved in solution. True. False Gaps in Geologic RecordAn causes gaps in the geologic record.Unconformities occur for two reasons:1. Rocks erode, and is deposited.2. No sediment is over a long timeAnswer choice for 1# deposited, sediment, uncomformity2# same answers3# same answers !!NEED URGENT MATH HELP!! Please help me this is due in like 30 min and I have no idea how to do it. I will name u brainliest and like ur answer and give a 5 stars and try to answer any question you have asked. Please help and ty! :) In a play, which element related to theme is introduced in act 1, built up in act 2, and resolved in act 3? Is 2 a root of the polynomial? x^4 + 2x^3 - 9x^2 - 2x + 8 A student looks through an eyepiece which magnifies by 10x and on objective lens 4x. What is the total magnification? Please help me with this one Whos tryna do this for me!!! :(